[:it]Maturità 2017: ottavo quesito[:]

[:it]

Vladimir Kush

Un dado ha la forma di un dodecaedro regolare con le facce numerate da 1 a 12. Il dado è truccato in modo che la faccia contrassegnata dal numero 3 si presenti con una probabilità doppia rispetto a ciascun’altra faccia. Determinare il valore di p in percentuale e calcolare la probabilità che in 5 lanci del dado la faccia 3 esca almeno 2 volte.

Prerequisiti

  • conoscere i teoremi della probabilità
  • conoscere la permutazioni con ripetizione
  • conoscere la probabilità di eventi indipendenti

Sviluppo

La somma della probabilità di tutti gli eventi mi dà sempre 1.

Nel caso specifico la probabilità che esca la faccia dall’1 al 12 escluso il 3 è p mentre 2p se esce il 3; ho 11 facce equiprobabili, per cui vale la seguente equazione:

(1)   \begin{gather*} 2p+11p=1 \\ 13p =1 \\ p=\cfrac{1}{13} \end{gather*}

Perché un evento possa verificarsi almeno significa che potrebbe sempre capitare o esserci almeno il numero di volte richiesto.

Nel caso specifico la probabilità che in 5 lanci esca il 3 è dato da:

(2)   \begin{equation*} \left (\cfrac{2}{13} \right )^5 \end{equation*}

Per conoscere tutte le possibili permutazioni nel caso in cui uscisse 4 volte il 3 ed una sola volta un altro numero uso la seguente relazione:

(3)   \begin{equation*} P_{n}^{h,k}=\cfrac{n!}{h!k!\cdot ...} \end{equation*}

Si usa ad esempio quante parole diverse non di senso compito possono essere fatte con la parola AABCDE con il gruppo AA che si ripete.

Perché esca 4 volte
ho

(4)   \begin{equation*} P_{5}^{4,1}=\cfrac{5!}{4!1!}=5 \end{equation*}

ad esempio 3 3 3 3 5

La probabilità diventa:

(5)   \begin{equation*} 5\left (\cfrac{2}{13} \right )^4\cfrac{11}{13} \right \end{equation*}

Perché esca 3 volte
ho

(6)   \begin{equation*} P_{5}^{3,2}=\cfrac{5!}{3!1!}=10 \end{equation*}

ad esempio 3 3 3 5 5

La probabilità diventa:

(7)   \begin{equation*} 10\left (\cfrac{2}{13} \right ) ^3\left (\cfrac{11}{13}\right )^2 \right \end{equation*}

Perché esca 2 volte
ho

(8)   \begin{equation*} P_{5}^{2,3}=\cfrac{5!}{2!3!}=10 \end{equation*}

ad esempio 3 3 5 5 5

La probabilità diventa:

(9)   \begin{equation*} 10\left (\cfrac{2}{13} \right )^2\left (\cfrac{11}{13}\right )^3 \right \end{equation*}

sommando la (2), (5), (7) e la (9) si ha

(10)   \begin{equation*} \left (\cfrac{2}{13} \right )^5+5\left (\cfrac{2}{13} \right )^4\cfrac{11}{13} \right +10\left (\cfrac{2}{13} \right ) ^3\left (\cfrac{11}{13}\right )^2 \right + 10\left (\cfrac{2}{13} \right )^2\left (\cfrac{11}{13}\right )^3 \right=0,1719 \end{equation*}

in percentuale diventa

il 17,19%[:]

Pubblicato in Senza categoria | Lascia un commento

[:it]Maturità 2017: settimo quesito[:]

[:it]

Alex Alemany

Determinare le coordinate dei centri delle sfere di raggio \sqrt{6} tangenti sl piano \pi di equazione:

(1)   \begin{equation*} x+2y-z+1=0 \end{equation*}

nel suo punto P di coordinate (1,0,2).

Prerequisiti

  • conoscere l’equazione della sfera nello spazio
  • conoscere la formula che esprime la distanza di un punto da un piano
  • conoscere l’equazione della retta passante per un punto e perpendicolare ad un piano
  • conoscere la condizione di appartenenza di un punto ad una retta

Sviluppo

L’equazione di una retta passante per un punto in forma parametrica è:

(2)   \begin{equation*} \left\{\begin{matrix} x=x_{0}+lt \\ y=y_{0}+mt\\ z=z_{0}+nt \end{matrix}\right. \end{equation*}

l,m,n, rappresentano le coordinate del vettore direzione v(l,m,n) ossia quello parallelo alla retta.

L’equazione generale di un piano ha equazione:

(3)   \begin{equation*} ax+by+cz+d=0 \end{equation*}

i coefficienti a,b e c rappresentano le coordinate del vettore perpendicolare al piano.

Unendo queste due richiami teorici la retta passante per P(1,0,2) e perpendicolare al piano \pi: x+2y-z+1=0 in forma parametrica è:

(4)   \begin{equation*} \left\{\begin{matrix} x=1+t \\ y=2t\\ z=2-t \end{matrix}\right. \end{equation*}

La formula della sfera è:

(5)   \begin{equation*} \left ( x-\alpha \right )^2+\left ( y-\beta \right )^2+\left ( z-\gamma \right )^2=r^{2} \end{equation*}

con C\left ( \alpha ,\beta ,\gamma \right ) centro della sfera ed r raggio della sfera.

applicandola si ha:

(6)   \begin{equation*} \left ( x-\alpha \right )^2+\left ( y-\beta \right )^2+\left ( z-\gamma \right )^2=6 \end{equation*}

Adesso richiamo la formula della distanza di un punto da un piano:

(7)   \begin{equation*} d=\cfrac{\left | ax_{0}+by_{0}+cz_{0}+d \right |}{\sqrt{a^{2}+b^{2}+c^{2}}} \end{equation*}

con a,b,c i coefficienti del piano (3) e P(x_{0},y_{0},z_{0}) il punto P di cui si vuole conoscere la distanza dal piano stesso.

Applicandola sapendo che la distanza tra il centro C\left ( \alpha ,\beta ,\gamma \right ) e il piano \pi: x+2y-z+1=0 vale \sqrt{6}:

(8)   \begin{equation*} d=\cfrac{\left | \alpha \cdot 1+\beta \cdot 2+\gamma \cdot -1 +1 \right |}{\sqrt{1+4+1}}}=\sqrt{6} \end{equation*}

Il centro C\left ( \alpha ,\beta ,\gamma \right ) appartiene alla retta trovata (4) per cui essa diventa:

(9)   \begin{equation*} \left\{\begin{matrix} \alpha =1+t \\ \beta =2t\\ \gamma=2-t \end{matrix}\right. \end{equation*}

adesso esprimo t in funzione delle coordinate del centro

(10)   \begin{equation*} \left\{\begin{matrix} t=\alpha-1 \\ \beta =2\alpha-2\\ \gamma=3-\alpha \end{matrix}\right. \end{equation*}

Adesso sostituisco i valori trovati nella (8) ed ho:

(11)   \begin{gather*} \cfrac{\left | \alpha+2(2\alpha-2)-3+\alpha+1 \right |}{\sqrt{1+4+1}}=\sqrt{6} \\ \left | 6\alpha -6 \right |=6 \\ \left | \alpha -1 \right |=1 \end{gather*}

Sapendo che in generale

(12)   \begin{equation*} \left | x \right |=\left\{\begin{matrix} x & x\geqslant 0\\ -x & x<0 \end{matrix}\right. \end{equation*}

la applico:

(13)   \begin{equation*} \left | \alpha -1 \right |=\left\{\begin{matrix} \alpha -1 & \alpha \geqslant 1\\ -\alpha +1 & \alpha <1 \end{matrix}\right. \end{equation*}

quindi ho due equazioni:

(14)   \begin{gather*} \alpha -1=1 \\ \alpha =2 \end{gather*}

e

(15)   \begin{gather*} -\alpha +1=1 \\ \alpha =0 \end{gather*}

i due centri hanno equazione:

(16)   \begin{equation*} C_{1}\left ( 0,-2,3 \right ) \end{equation*}

e

(17)   \begin{equation*} C_{2}\left ( 2,2,1 \right ) \end{equation*}

[:]

Pubblicato in Senza categoria | Lascia un commento

[:it]Maturità 2017: sesto quesito[:]

[:it]

Alex Alemany

Determinare il numero reale a in modo che il valore di

(1)   \begin{equation*} \underset{x\rightarrow 0}{lim}\cfrac{\sin (x)-x}{x^{a}} \end{equation*}

sia un numero reale non nullo.

Prerequisiti

  • conoscere come calcolare un limite
  • conoscere i limiti indeterminati
  • conoscere il teorema di De l’Hopital

Sviluppo

Se sostituisco il valore a cui tende la x nella (1) mi accorgo si essere nella condizione \cfrac{0}{0} e posso applicare De l’Hopital.

Il teorema di De l’Hopital afferma che nel caso in cui ci si trova nella condizione \cfrac{0}{0} o \cfrac{\infty }{\infty } allora:

(2)   \begin{equation*} \underset{x\rightarrow x_{0}}{lim}\cfrac{f(x)}{g(x)}=\underset{x\rightarrow x_{0}}{lim}\cfrac{f'(x)}{g'(x)} \end{equation*}

che significa fare la derivata del numeratore e del denominatore separatamente e NON la derivata del quoziente di funzione!

Applico la (2):

(3)   \begin{equation*} \underset{x\rightarrow 0}{lim}\cfrac{\sin (x)-x}{x^{a}}=\underset{x\rightarrow 0}{lim}\cfrac{\cos (x)-1}{a\cdot x^{a-1}} \end{equation*}

Sostituisco il valore a cui tende la x nella (3) e mi accorgo si essere ancora nella condizione \cfrac{0}{0} ma potrei porre l’esponente della x del denominatore a 0 e conseguentemente togliere la condizione che lo annulla (a=1), in questo caso il limite tenderebbe a 0, ma stiamo cercando un valore reale non nullo.
Applico nuovamente De l’Hopital.

(4)   \begin{equation*} \underset{x\rightarrow 0}{lim}\cfrac{\cos (x)-1}{a\cdot x^{a-1}}= \underset{x\rightarrow 0}{lim}\cfrac{-\sin (x)}{a\cdot (a-1) \cdot x^{a-2}} \end{equation*}

Sostituisco il valore a cui tende la x nella (4) e mi accorgo si essere ancora nella condizione \cfrac{0}{0} ma potrei porre l’esponente della x del denominatore a 0 e conseguentemente togliere la condizione che lo annulla (a=2), in questo caso il limite tenderebbe a 0, ma stiamo cercando un valore reale non nullo.
Applico nuovamente De l’Hopital.

(5)   \begin{equation*} \underset{x\rightarrow 0}{lim}\cfrac{-\sin (x)}{a\cdot (a-1) \cdot x^{a-2}}=\underset{x\rightarrow 0}{lim}\cfrac{-\cos (x)}{a\cdot (a-1) \cdot (a-2) \cdot x^{a-3}} \end{equation*}

Sostituisco il valore a cui tende la x nella (4) e mi accorgo di essere adesso nella condizione \cfrac{-1}{0}=\infty ma pongo l’esponente della x del denominatore a 0 e, conseguentemente, togliere la condizione che lo annulla (a=3), in questo caso la (5) diventa:

(6)   \begin{equation*} \underset{x\rightarrow 0}{lim}\cfrac{-\cos (x)}{a\cdot (a-1) \cdot (a-2) \cdot x^{a-3}}=\underset{x\rightarrow 0}{lim}\cfrac{-\cos (0)}{3\cdot (3-1) \cdot (3-2) \cdot x^{0}}=-\cfrac{1}{6} \end{equation*}

L’unico valore per cui il limite assume un valore reale non nullo è:

(7)   \begin{equation*} a=3 \end{equation*}

[:]

Pubblicato in Senza categoria | Lascia un commento

[:it]Maturità 2017: quesito 5[:]

[:it]

Alex Alemany

Dati i punti A(-2,3,1), B(3,0,-1), C(2,2,-3), determinare l’equazione della retta r passante per A e per B e l’equazione del piano \pi perpendicolare ad r e passante C.

Prerequisiti

  • conoscere l’equazione della retta passante per due punti nello spazio
  • capire il significato dei coefficienti numerici della retta e di un piano
  • capire il significato di un punto appartenente ad una curva

Sviluppo

L’equazione di una retta passante per due punti ha equazione:

(1)   \begin{equation*} \left\{\begin{matrix} x=x_{2}+\left ( x_{1}-x_{2} \right )t \\ y=y_{2}+\left ( y_{1}-y_{2} \right )t\\ z=z_{2}+\left ( z_{1}-z_{2} \right )t \end{matrix}\right. \end{equation*}

Applicandola al caso posto dal quesito, la retta passante per A e per B ha equazione:

(2)   \begin{equation*} \left\{\begin{matrix} x=-2+5t \\ y=3-3t\\ z=1-2t \end{matrix}\right. \end{equation*}

i coefficienti di x (5), di y (-3), di z (-2), rappresentano le coordinate del vettore direzione v(5,-3,-2) ossia quello parallelo alla retta.
L’equazione generale di un piano ha equazione:

(3)   \begin{equation*} ax+by+cz+d=0 \end{equation*}

i coefficienti a,b e c rappresentano le coordinate del vettore perpendicolare al piano.

Conseguenza di questo l’equazione del piano \pi utilizza le coordinate della retta:

(4)   \begin{equation*} 5x-3y-2z+d=0 \end{equation*}

Per trovare d è sufficiente sostituire le coordinate del punto C e risolvere la relativa equazione di primo grado in d:

(5)   \begin{gather*} 10-6+6+d=0 \\ d=-10 \end{gather*}

L’equazione del piano risulta:

(6)   \begin{equation*} 5x-3y-2z-10=0 \end{equation*}

[:]

Pubblicato in Senza categoria | Lascia un commento

[:it]Maturità 2017: quarto quesito[:]

[:it]

Samy Charnine

Per sorteggiare numeri reali nell’intervallo [0;2] viene realizzato un generatore di numeri casuali che fornisce numeri distribuiti, in tale intervallo, con densità di probabilità data dalla funzione:

(1)   \begin{equation*} f(x)=\cfrac{3}{2}x^{2}-\cfrac{3}{4}x^{3} \end{equation*}

Quale sarà il valore medio dei numeri generati?
Qual è la probabilità che il primo numero estratto sia \cfrac{4}{3}?
Qual è la probabilità che il secondo numero estratto sia minore di 1?

Prerequisiti

  • conoscere cosa rappresenta in ambito statistico la densità di probabilità
  • conoscere come calcolare il valor medio di una variabile aleatoria continua
  • saper sviluppare un integrale

Sviluppo

La densità di probabilità fornisce, come dice il nome stesso, la probabilità che accada un evento continuo all’interno di un intervallo. Per conoscere tale valore si deve calcolare l’area sottesa dalla curva ed essa si calcola attraverso proprio un integrale nell’intervallo voluto.

(2)   \begin{equation*} P\left ( X\in A \right )=\int_{A}p_{X}(x)dx \end{equation*}

Per calcolare il valor medio di una variabile aleatoria continua si deve applicare:

(3)   \begin{equation*} m=\int_{A}xf(x)dx \end{equation*}

applicandolo al caso specifico il valor medio risulta

(4)   \begin{gather*} m=\int_{0}^{2}\cfrac{3}{2}x^{3}-\cfrac{3}{4}x^{4}dx \\ \left\begin{matrix} \cfrac{3}{2}\cfrac{x^{4}}{4}-\cfrac{3}{4}\cfrac{x^5}{5} \end{matrix}\right|_{0}^{2} \\ \cfrac{3}{2} \cdot \cfrac{16}{4}-\cfrac{3}{4}\cdot \cfrac{32}{5} \\ 6-\cfrac{24}{5}= \\ m=\cfrac{6}{5} \end{gather*}

Chiedere la probabilità che venga estratto un particolare numero non ha significato in quanto essendo definito uno spazio continuo di valori cercare di sapere esattamente un valore preciso è come cercare di dare una descrizione statistica di eventi discreti con una densità di probabilità.

Mentre si può calcolare agevolmente la probabilità che un numero estratto sia minore di 1 sviluppando il seguente integrale:

(5)   \begin{gather*} P[x<1]=\int_{0}^{1}\cfrac{3}{2}x^{2}-\cfrac{3}{4}x^{3}dx \\ \left\begin{matrix} \cfrac{3}{2}\cfrac{x^{3}}{3}-\cfrac{3}{4}\cfrac{x^4}{4} \end{matrix}\right|_{0}^{1} \\ \cfrac{3}{2} \cdot \cfrac{1}{3}-\cfrac{3}{4}\cdot \cfrac{1}{4} \\ \cfrac{1}{2}-\cfrac{3}{16}= \\ P[x<1]=\cfrac{5}{16} \end{gather*}

[:]

Pubblicato in Senza categoria | Lascia un commento

[:it]Maturità 2017: terzo quesito[:]

[:it]

Paul Klee

Sapendo che:

(1)   \begin{equation*} \underset{x\rightarrow0}{lim}\cfrac{\sqrt{ax+2b}-6}{x}=1$ \end{equation*}

determinare i valori di a e b.

Prerequisiti

  • conoscere il calcolo con i limiti
  • sapere fare la razionalizzazione inversa
  • prodotto notevole della differenza del binomio
  • conoscere il metodo della fattorizzazione per eliminare gli zeri del numeratore e denominatore

Sviluppo

Sostituendo il valore 0 alla x del numeratore e del denominatore mi trovo nella situazione:

\cfrac{\sqrt{2b}-6}{0}=\infty

La razionalizzazione inversa è necessaria per poter semplificare la x presente al numeratore con quella del denominatore; utilizzo il prodotto notevole

(2)   \begin{equation*} \left ( a+b \right )\cdot \left ( a-b \right )=a^{2}-b^{2} \end{equation*}

Bisogna anche ricordarsi che il quadrato di una radice quadrata mi dà proprio il radicando ossia l’argomento della radice.

(3)   \begin{equation*} \left (\sqrt{5}  \right )^2=5 \end{equation*}

Applicandola ad un prodotto notevole ho:

(4)   \begin{equation*} \left ( \sqrt{a}+b \right )\left ( \sqrt{a}-b \right )=\left ( \sqrt{a} \right )^2-b^{2}=a-b^2 \end{equation*}

Faccio la razionalizzazione inversa ossia moltiplico il numeratore ed il denominatore per \sqrt{ax+2b}+6 e la (1) diventa:

(5)   \begin{gather*} \cfrac{\left ( \sqrt{ax+2b}-6 \right )\left ( \sqrt{ax+2b}+6 \right )}{x\left ( \sqrt{ax+2b}+6 \right )} \\ \cfrac{ ax+2b-36 }{x\left ( \sqrt{ax+2b}+6 \right )} \end{gather*}

Adesso per togliere lo zero che annulla sia il numeratore che il denominatore pongo:

(6)   \begin{gather*} 2b-36=0 \\ b=18 \end{gather*}

sostituendo il valore trovato nella (6) nella (1) il limite diventa:

(7)   \begin{equation*} \underset{x\rightarrow0}{lim}\cfrac{ ax }{x\left ( \sqrt{ax+36}+6 \right )} \\ \end{equation*}

Solo adesso, dopo avere eseguito la razionalizzazione inversa, posso semplificare la x presente nel numeratore con quella del denominatore.

(8)   \begin{equation*} \underset{x\rightarrow0}{lim}\cfrac{ a }{\sqrt{ax+36}+6}  \end{equation*}

Perché tale limite vada ad 1 è sufficiente risolvere la seguente equazione avendo posto a 0 il valore dell x (il valore a cui tende il limite)

(9)   \begin{gather*} \cfrac{ a }{\sqrt{36}+6}=1 \\ \cfrac{ a }{6+6}=1 \\ a=12 \end{gather*}

quindi ricapitolando i due valori sono:

(10)   \begin{gather*} a=12 \\ b=18 \end{gather*}

[:]

Pubblicato in Senza categoria | Contrassegnato | Lascia un commento

[:it]Maturità 2017: secondo quesito[:]

[:it]

Andrea Wan

Una torta di forma cilindrica è collocata sotto una cupola di plastica di forma semisferica. Dimostrare che la torta occupa meno dei 3/5 del volume della semisfera.

Prerequisiti

  • conoscere il volume del cilindro e della sfera
  • trigonometria e disequazioni trigonometriche
  • studio di funzione

Sviluppo

(1)   \begin{equation*} V_{torta}<\cfrac{3}{5}\cfrac{V_{sfera}}{2} \end{equation*}

Schematizzo il problema in questa maniera:

(2)   \begin{gather*}  \pi r^{2} h<\cfrac{3}{5}\cdot \cfrac{4}{3}\cdot \cfrac{1}{2}\pi R^{3} \\  r^{2} h<\cfrac{2}{5}R^{3} \end{gather*}

(3)   \begin{gather*} h=R\sin \alpha \\ r=R\cos \alpha \\ \end{gather*}

sostituendo questo nella (2) devo dimostrare che:

(4)   \begin{gather*} R^{2}\cos^{2} \alpha \cdot R\sin \alpha < \cfrac{2}{5}R^{3} \\ \cos^{2} \alpha \cdot \sin \alpha < \cfrac{2}{5}\\ \left ( 1-\sin ^{2}\alpha  \right )\cdot \sin \alpha < \cfrac{2}{5}\\ \sin \alpha -\sin ^{3}\alpha -\cfrac{2}{5}< 0 \end{gather*}

pongo \sin \alpha =t
con 0\leqslant \alpha \leqslant \cfrac{\pi}{2} e quindi 0\leqslant t \leqslant 1

devo risolvere:

(5)   \begin{gather*} 5t-5t^3-2< 0 \\ 5t^3-5t+2> 0 \\ \end{gather*}

Effettuando uno studio di funzione di:

(6)   \begin{equation*} y=5t^3-5t+2 \end{equation*}

vedo che ha questo andamento per cui ho dimostrato l’affermazione del problema in quanto la funzione è sempre positiva nell’intervallo considerato

Rendered by QuickLaTeX.com

[:]

Pubblicato in Senza categoria | Lascia un commento

[:it]Maturità 2017: problema 2[:]

[:it]

Consideriamo la funzione f:\mathbb{R}\rightarrow \mathbb{R}, periodica di periodo T=4 il cui grafico, nell’intervallo  [0;4], è il seguente:

Come si evince dalla figura 1, i tratti OB, BD, DE del grafico sono i segmenti i cui estremi hanno coordinate: O(0,0), B(1,1), D(3,1), E(4,0).

1) Stabilisci in quali punti del suo insieme di definizione la funzione f è continua e in quali è derivabile e verifica l’esistenza dei limiti: \underset{x\rightarrow \infty }{lim} f(x) e \underset{x\rightarrow \infty }{lim} \cfrac{f(x)}{x}; qualora esistano, determinare il valore.
Rappresenta inoltre, per x\in \left [ 0;4 \right ], i grafici delle funzioni:

(1)   \begin{equation*} g(x)=f'(x) \end{equation*}

(2)   \begin{equation*} h(x)=\int_{0}^{x}f(t)dt \end{equation*}

Prerequisiti

  • conoscere le caratteristiche dei punti angolosi
  • retta passante per due punti
  • conoscere i limiti
  • conoscere bene le derivate
  • conoscere gli integrali
  • saper fare un grafico con sicurezza delle funzioni elementari quali retta e parabola

Svolgimento
\underset{x\rightarrow \infty }{lim} f(x)
non può esistere essendo f(x) una funzione che oscilla tra +1 e -1.
Invece
\underset{x\rightarrow \infty }{lim} \cfrac{f(x)}{x}=0
in quanto f(x) varia sempre da +1 a -1 e diviso per un numero che tende all’infinito inevitabilmente si avrà lo 0 come risultato.

Per risolvere la (1), preferisco prima scrivere le equazioni che rappresentano la funzione periodica f(x).

L’equazione della retta passante per O(0,0), B(1,1) è proprio la bisettrice del primo quadrante y=x.
Per avere l’equazione della retta passante per B(1,1), D(3,-1) uso la relazione:

(3)   \begin{equation*} \cfrac{y-y_{D} }{ y_{B}-y_{D} }=\cfrac{x-x_{D}}{x_{B}-x_{D}} \end{equation*}

(4)   \begin{gather*} \cfrac{y+1}{1+1}=\cfrac{x-3}{1-3} \\ y=-x+2 \end{gather*}

Per avere l’equazione della retta passante per D(3,-1), E(4,0) uso la relazione (3)

(5)   \begin{gather*} \cfrac{y-0}{-1+0}=\cfrac{x-4}{3-4} \\ y=x-4 \end{gather*}

Ricapitolando:

(6)   \begin{equation*} f(x)=\left\{\begin{matrix} x & 0\leqslant x\leqslant 1\\ -x+2 &1\leqslant x\leqslant 3 \\ x-4 & 3\leqslant x\leqslant 4 \end{matrix}\right. \end{equation*}

La (1) adesso si può calcolare derivando la (6) tra gli opportuni intervalli:

(7)   \begin{equation*} g(x)=f'(x)=\left\{\begin{matrix} 1 & 0\leqslant x\leqslant 1\\ -1 &1\leqslant x\leqslant 3 \\ 1 & 3\leqslant x\leqslant 4 \end{matrix}\right. \end{equation*}

La sua rappresentazione grafica è:

Rendered by QuickLaTeX.com

Per trovare la (2), uso ancora la (6) considerando gli intervalli opportuni.

Per x\in [0;1]

(8)   \begin{equation*} h(x)=\int_{0}^{x}tdt=\left\begin{matrix} \cfrac{1}{2}t^{2}\end{matrix}\right|_{0}^{x}=\cfrac{x^{2}}{2} \end{equation*}

Per x\in [1;3]

(9)   \begin{gather*} h(x)=\int_{0}^{x}f(t)dt=\int_{0}^{1}tdt+\int_{1}^{x}(-t+2)dt= \\ \left\begin{matrix} \cfrac{1}{2}t^{2}\end{matrix}\right|_{0}^{1}+\left\begin{matrix} -\cfrac{t^{2}}{2}+2t\end{matrix}\right|_{1}^{x}= \\ \cfrac{1}{2}-\cfrac{x^{2}}{2}+2x+\cfrac{1}{2}-2=\\ -1-\cfrac{x^{2}}{2}+2x \end{gather*}

Per x\in [3;4]

(10)   \begin{gather*} h(x)=\int_{0}^{x}f(t)dt=\int_{0}^{1}tdt+\int_{1}^{3}(-t+2)dt+\int_{3}^{x}(t-4)dt \\ \left\begin{matrix} \cfrac{1}{2}t^{2} \end{matrix}\right|_{0}^{1}+ \left\begin{matrix} -\cfrac{t^{2}}{2}+2t \end{matrix}\right|_{1}^{3}+ \left\begin{matrix} (\cfrac{t^{2}}{2}-4t) \end{matrix}\right|_{3}^{x}= \\ \cfrac{1}{2}+\left ( -\cfrac{9}{2}+6 \right )-\left ( -\cfrac{1}{2}+2 \right )+\cfrac{x^2}{2}-4x-\cfrac{9}{2}+12\\ 8+\cfrac{x^2}{2}-4x \end{gather*}

Riunisco la (8) la (9) e la (10)

(11)   \begin{equation*} h(x)=\left\{\begin{matrix} \cfrac{x^{2}}{2} & 0\leqslant x\leqslant 1\\ -1-\cfrac{x^{2}}{2}+2x& 1\leqslant x\leqslant 3\\ 8+\cfrac{x^2}{2}-4x & 3\leqslant x\leqslant 4 \end{matrix}\right. \end{equation*}

La sua rappresentazione grafica è la rappresentazione di tre parabole negli opportuni intervalli:

Rendered by QuickLaTeX.com

2) Considera la funzione:

(12)   \begin{equation*} s(x)=\sin \left ( bx \right ) \end{equation*}

con b costante reale positiva; determina b in modo che s(x) abbia lo stesso periodo di f(x).
Dimostra che la porzione quadrata di piano OABC in figura 1 viene suddivisa dai grafici di f(x) e s(x) in tre parti distinte e determina le probabilità che un punto preso a caso all’interno del quadrato OABC ricada in ciascuna delle 3 parti individuate.

Prerequisiti

  • conoscere il significato di periodicità di una funzione trigonometrica
  • conoscere la definizione di probabilità
  • applicare il concetto di probabilità nel caso della geometria piana
  • conoscere l’integrale come area di una regione finita di piano

Sviluppo

Perché la funzione (12) abbia periodo 4 è sufficiente studiare quando:

(13)   \begin{equation*} \sin \left ( bx \right )=0 \end{equation*}

esso si annullerà quando:

(14)   \begin{gather*} bx=\pi \\ bx=2\pi \end{gather*}

avendo periodo 4 significa che sia per x=4 ma anche x=2 si annulla per cui:

(15)   \begin{gather*} b2=\pi \\ b4=2\pi \end{gather*}

entrambe mi portano ad affermare che

(16)   \begin{equation*} b=\cfrac{\pi }{2} \end{equation*}

Rappresento graficamente la situazione per x\in [0;1], ossia all’interno del quadrato OABC.

Rendered by QuickLaTeX.com

e si osserva che si hanno tre regioni.
La definizione di probabilità è:

(17)   \begin{equation*} P=\cfrac{favorevoli}{probabili} \end{equation*}

In questo caso: l’evento favorevole è una delle tre aree, l’evento probabile è l’area totale che vale 1 essendo un quadrato di lato 1.

L’area della zona superiore si calcola sottraendo all’area del quadrato di lato unitario l’integrale della funzione s(x):

(18)   \begin{gather*} A_{1}=1-\int_{0}^{1}\sin \left ( \cfrac{\pi}{2}x \right )dx= \\ 1-\left\begin{matrix} \left (-\cos \left ( \cfrac{\pi}{2}x \right )\cdot \cfrac{2}{\pi} \right ) \end{matrix}\right|_{0}^{1}= \\ 1-\cfrac{2}{\pi} \end{gather*}

Essa corrisponde alla probabilità che il punto possa cadere in quest’area.

L’area della zona centrale si calcola attraverso l’integrale della funzione s(x) a cui sottraggo l’area del triangolo di base 1 ed uguale altezza.

(19)   \begin{gather*} A_{2}=\int_{0}^{1}\sin \left ( \cfrac{\pi}{2}x \right )dx-\cfrac{1}{2}= \\ \left\begin{matrix} -\cos \left ( \cfrac{\pi}{2}x \right )\cdot \cfrac{2}{\pi} -\cfrac{1}{2} \end{matrix}\right|_{0}^{1}= \\ \cfrac{2}{\pi}-\cfrac{1}{2} \end{gather*}

La terza area corrisponde all’area del triangolo di base 1 ed altezza unitaria:

(20)   \begin{equation*} A_{3}=\cfrac{1}{2} \end{equation*}

3) Considerando ora le funzioni:

(21)   \begin{equation*} f(x)^2 \end{equation*}

e

(22)   \begin{equation*} s(x)^2 \end{equation*}

discuti, anche con argomentazioni qualitative, le variazioni (in aumento o in diminuzione) dei tre valori di probabilità determinati al punto precedente

Prerequisiti

  • conoscenza del grafico del quadrato di una funzione e sua approssimazione grafica
  • sapere sviluppare in maniera molto sicura gli integrali

Sviluppo

Tra 0\leqslant x\leqslant 1 la f(x)^2 da retta diventa una parabola.
mentre la funzione trigonometrica s(x) ha un flesso per x=\cfrac{1}{2}.
Si può quindi qualitativamente dire che
A_{3} diminuisce
A_{1} aumenta

Per meglio evidenziare la situazione faccio il grafico delle tre curve che può essere ricavato solo se precedentemente ci si è esercitati con i grafici delle funzioni trigonometriche.

Rendered by QuickLaTeX.com

Per l’area compresa si deve calcolare il seguente integrale:

(23)   \begin{equation*} A_{2}=\int_{0}^{1}\left (\sin \left ( \cfrac{\pi}{2}x \right ) \right )^2-x^{2}dx \end{equation*}

Vi sono due strade e la migliore è analizzare il grafico:

Rendered by QuickLaTeX.com

e si nota che l’area del grafico della funzione trigonometrica è proprio l’area del triangolo di base unitaria ed altezza uguale in quanto i contributi della “gobba” si bilanciano.

Quindi è sufficiente calcolare il seguente integrale notevolmente più semplice:
Per l’area compresa si deve calcolare il seguente integrale:

(24)   \begin{gather*} A_{2}=\cfrac{1}{2}-\int_{0}^{1}x^{2}dx= \\ \cfrac{1}{2}-\left\begin{matrix} \cfrac{x^3}{3} \end{matrix}\right|_{0}^{1}= \\ \cfrac{1}{2}-\cfrac{1}{3}=\cfrac{1}{6} \end{gather*}

valore di poco inferiore a quello precedente.

4) Determina il volume del solido generato dalla rotazione attorno all’asse y della porzione di piano compresa tra il grafico della funzione h(x) per x\in [0;3] e l’asse x.

Prerequisiti

  • conoscere la formula per il calcolo del volume di figure piane che ruotano attorno all’asse y.

Sviluppo

Si utilizza la seguente formula:

(25)   \begin{equation*} V=2\pi\int_{a}^{b}xf(x)dx \end{equation*}

La applico ad h(x)

(26)   \begin{gather*} V=2\pi\int_{0}^{1}\cfrac{x^3}{2}dx+\int_{1}^{3}-x-\cfrac{x^3}{2}+2x^{2}dx= \\ 2\pi \cdot \left\begin{matrix} \cfrac{x^4}{8} \end{matrix}\right|_{0}^{1}+ \left\begin{matrix} -\cfrac{x^2}{2}-\cfrac{x^4}{8}+\cfrac{2x^3}{3}- \end{matrix}\right|_{1}^{3}= \\ 2\pi \cdot \left (\cfrac{1}{8} -\cfrac{9}{2}-\cfrac{81}{8}+\cfrac{54}{3}+\cfrac{1}{2}+\cfrac{1}{8}-\cfrac{2}{3} \right )= \\ 2 \pi\cdot \cfrac{83}{24}= \pi\cdot \cfrac{83}{12} \end{gather*}

[:]

Pubblicato in Senza categoria | Lascia un commento

[:it]Maturità 2017: problema 1[:]

[:it]

Si può pedalare agevolmente su una bicicletta a ruote quadrate? A New York, al MoMath-Museum of Mthematics si può fare, in uno dei padiglioni dedicati al divertimento matematico

E’ però necessario che il profilo della pedana su cui il lato della ruota può scorrere soddisfi alcuni requisiti.

nella figura 2 è riportata una rappresentazione della situazione nel piano cartesiano Oxy: il quadrato di lato DE=2 (in opportune unità di misura) e di centro C rappresenta la ruota della bicicletta, il grafico della funzione f(x) rappresenta il profilo della pedana.

1) Sulla base delle informazioni ricavabili dal grafico in figura 2, mostra, con le opportune argomentazioni, che la funzione:

f(x)=\sqrt{2}-\cfrac{e^{x}+e^{-x}}{2} x\in \mathbb{R}
rappresenta adeguatamente il profilo della pedana per x\in \left [ -a;a \right ]; determina inoltre il valore degli estremi a e -a dell’intervallo.

Prerequisiti

  • saper risolvere un’equazione esponenziale
  • conoscere il logaritmo e le sue proprietà
  • saper derivare una funzione esponenziale
  • calcolare il massimo di una funzione
  • saper risolvere una disequazione esponenziale

Sviluppo

Calcolo le intersezioni della funzione annullandola

(1)   \begin{gather*} \sqrt{2}-\cfrac{e^{x}+e^{-x}}{2}=0 \\ 2\sqrt{2}-e^{x}-e^{-x}=0 \\ 2\sqrt{2}-e^{x}-\cfrac{1}{e^{x}}=0 \\ 2\sqrt{2}e^{x}-e^{2x}-1=0 \\ e^{2x}-2\sqrt{2}e^{x}+1=0 \\ \end{gather*}

adesso pongo

(2)   \begin{equation*} e^{x}=t \end{equation*}

e devo risolvere una semplice equazione di secondo grado:

(3)   \begin{gather*} t^2-2\sqrt{2}t+1=0 \\ t_{1,2}=\cfrac{2\sqrt{2}\pm \sqrt{8-4}}{2}=\cfrac{2\sqrt{2}\pm 2}{2}=\sqrt{2}\pm 1 \end{gather*}

torno alla (2) con le soluzioni trovate e risolvo la relativa equazione esponenziale

(4)   \begin{gather*} e^{x}=\sqrt{2}+1 \\ x=\ln (\sqrt{2}+1)\simeq 0.88 \end{gather*}

La prima intersezione è:

a\simeq 0.88

(5)   \begin{gather*} e^{x}=\sqrt{2}-1 \\ x=\ln (\sqrt{2}-1)\simeq -0.88 \end{gather*}

La seconda intersezione è:

a\simeq -0.88

Adesso calcolo la derivata prima e determino i punti di massimo o di minimo così da verificare che l’andamento della curva sia come quello della figura 2.

f(x)=\sqrt{2}-\cfrac{e^{x}+e^{-x}}{2}

(6)   \begin{equation*} f(x)^{'}=-\cfrac{e^{x}-e^{-x}}{2} \end{equation*}

La annullo per verificare i potenziali punti di massimo o minimo

(7)   \begin{equation*} -\cfrac{e^{x}-e^{-x}}{2}=0 \end{equation*}

il metodo più veloce per risolverla è la loro rappresentazione grafica che permette poi anche di studiare subito il segno per determinare dove la funzione cresce o decresce.

Rendered by QuickLaTeX.com

e si nota subito che l’unico punto in cui si intersecano è lo 0.
Tale punto è di massimo o di minimo?
Studio il segno della derivata prima.

sempre dal grafico si nota che e^x è maggiore di e^{-x} per x>0 ma essendoci il segno meno davanti alla derivata prima si ha il seguente grafico del segno:

Rendered by QuickLaTeX.com

e quindi per x=0 si ha proprio un punto di massimo,

f(0)=\sqrt{2}-\cfrac{e^{0}+e^{^{-0}}}{2}=\sqrt{2}-1\simeq 0.41

ed il grafico della funzione è proprio quello della figura 2.

Per visualizzare il profilo completo della pedana sulla quale la bicicletta potrà muoversi, si affiancano varie copie del grafico della funzione f(x) relativo all’intervallo [-a;a], come mostrato in figura 3.

2) Perché la bicicletta possa procedere agevolmente sulla pedana è necessario che:

  • a sinistra e a destra dei punti di non derivabilità i tratti del grafico siano ortogonali;
  • la lunghezza del lato della ruota quadrata risulti pari alla lunghezza di una “gobba”, cioè dell’arco di curva di equazione y=f(x) per x\in \left [ -a;a \right ]

stabilisci se tali condizioni sono verificate. In generale, la lunghezza dell’arco di curva avente equazione y=\varphi (x) compreso tra le ascisse x_{1} e x_{2} è data da

(8)   \begin{equation*} \int_{x_{1}}^{x_{2}}\sqrt{1+\left ( \varphi '(x) \right )^2} dx \end{equation*}

Prerequisiti

  • riconoscere i punti di non derivabilità di una funzione
  • essere in grado di fare la derivata
  • collegamento tra il valore della derivata e l’inclinazione della retta tangente in quel punto.
  • essere in grado di sviluppare un integrale definito
  • proprietà dei logaritmi
  • ottima manualità con i radicali e i prodotti notevoli con i radicali stessi
  • molto precisi ed attenzione nei segni

Sviluppo

I punti di non derivabilità sono quelli in cui la curva interseca l’asse delle ascisse in quanto il valore della derivata destra e della derivata sinistra è differente. Nel caso specifico però calcolando la derivata in quei due punti posso avere il coefficiente angolare delle rette tangenti. Se esse sono tra loro opposte e reciproche allora i tratti del grafico di non derivabilità sono ortogonali.

Avevo già calcolato la derivata in (6) e risulta:

f(x)^{'}=-\cfrac{e^{x}-e^{-x}}{2}

La calcolo nei punti di intersezione già precedentemente trovati in (4) e (5) ossia:

x_{1}=\ln (\sqrt{2}-1)

x_{2}=\ln (\sqrt{2}+1)

(9)   \begin{equation*} f'(\ln (\sqrt{2}-1))=-\cfrac{e^{\ln (\sqrt{2}-1)}-e^{-\ln (\sqrt{2}-1)}}{2} \end{equation*}

ma partendo dalla definizione stessa di logaritmo

(10)   \begin{gather*} a^{b}=c \\ b=\log _{a}c \\ \end{gather*}

(11)   \begin{gather*} e^{\ln b}=c \\ \ln b=\ln c \\ b=c \end{gather*}

quindi

(12)   \begin{equation*} e^{\ln b}=b \end{equation*}

ed utilizzo anche la seguente proprietà:

(13)   \begin{equation*} c\log _{a}b=\log _{a}b^{c} \end{equation*}

che applicata in questo caso diventa:

(14)   \begin{equation*} -1\log _{a}b=\log _{a}b^{-1} \end{equation*}

Utilizzo la (12) e la (14) nella (9) e diventa:

(15)   \begin{gather*} -\cfrac{e^{\ln (\sqrt{2}-1)}-e^{-\ln (\sqrt{2}-1)}}{2}= \\ -\cfrac{\sqrt{2}-1-\cfrac{1}{\sqrt{2}-1}}{2}= \\ \cfrac{\left (\sqrt{2}-1 \right )^{2}-1}{2\left ( \sqrt{2}-1 \right )}= \\ \cfrac{2-2\sqrt{2}+1-1}{2\left ( \sqrt{2}-1 \right )}= \\ 1 \end{gather*}

seguendo lo stesso procedimento per l’altro punto d’intersezione

(16)   \begin{equation*} f'(\ln (\sqrt{2}+1))=-\cfrac{e^{\ln (\sqrt{2}+1)}-e^{-\ln (\sqrt{2}+1)}}{2}=-1 \end{equation*}

che forniscono il valore del coefficiente angolare delle rette tangenti e perché siano ortogonali deve valere la relazione:
m=-\cfrac{1}{m^{'}}

ed infatti è soddisfatta:

(17)   \begin{equation*} f'(\ln (\sqrt{2}-1))=-1=-\frac{1}{f'(\ln (\sqrt{2}+1))}=-\cfrac{1}{1}=-1 \end{equation*}

Nella seconda parte del quesito si chiedeva di calcolare un semplice integrale definito per calcolare la lunghezza dell’arco di curva, utilizzando il suggerimento, e porlo uguale alla lunghezza del quadrato che dato dal testo e vale 2.
Analiticamente deve valere questa condizione:

(18)   \begin{equation*} \int_{\ln \left ( \sqrt{2}-1 \right )}^{\ln \left ( \sqrt{2} +1\right )}\sqrt{1+\left ( -\cfrac{e^x-e^{-x}}{2} \right )^2}=2 \end{equation*}

Sviluppo l’argomento della radice

(19)   \begin{gather*} 1+\left ( -\cfrac{e^x-e^{-x}}{2} \right )^2= \\ 1+\cfrac{e^{2x}-2+e^{-2x}}{4}= \\ \cfrac{4+e^{2x}-2+e^{-2x}}{4}= \\ \cfrac{e^{2x}+2+e^{-2x} }{4}= \\ \cfrac{\left ( e^{x}+e^{-x} \right )^{2}}{4} \\ \end{gather*}

e l’integrale definito diventa:

(20)   \begin{equation*} \int_{\ln \left ( \sqrt{2}-1 \right )}^{\ln \left ( \sqrt{2} +1\right )} \sqrt{1+\left ( -\cfrac{e^x-e^{-x}}{2} \right )^2}=\int_{\ln \left ( \sqrt{2}-1 \right )}^{\ln \left ( \sqrt{2} +1\right )} \cfrac{1}{2}{\left ( e^{x}+e^{-x} \right ) \end{equation*}

la cui primitiva diventa:

(21)   \begin{equation*} \left\begin{matrix} \cfrac{1}{2}{\left ( e^{x}-e^{-x} \right )\end{matrix}\right|_{\sqrt{2}-1}^{\sqrt{2} +1} \end{equation*}

utilizzando tutte le proprietà dalla (10) alla (14) l’integrale definito diventa:

(22)   \begin{gather*} \cfrac{1}{2}\left [ \sqrt{2}+1- \cfrac{1}{\sqrt{2}+1}- \left ( \sqrt{2}-1-\cfrac{1}{\sqrt{2}-1} \right )\right ] \\ \cfrac{1}{2}\left [ \sqrt{2}+1- \cfrac{1}{\sqrt{2}+1}- \sqrt{2}+1+\cfrac{1}{\sqrt{2}-1} \right ] \\ \cfrac{1}{2}\left [ 2-\cfrac{1}{\sqrt{2}+1}+\cfrac{1}{\sqrt{2}-1} \right ] \\ \cfrac{1}{2}\left [ \cfrac{2\left ( \sqrt{2}+1 \right )\left ( \sqrt{2}-1 \right )-\sqrt{2}+1+\sqrt{2}+1}{\left ( \sqrt{2}+1 \right )\left ( \sqrt{2}-1 \right )} \right ] \\ \cfrac{1}{2}\left [ \cfrac{2+2}{1} \right ]=2 \end{gather*}

ed ho verificato l’affermazione del problema.

3) Consideriamo la similitudine dei triangoli rettangoli ACL e ALM in figura 4, e ricordando il significato geometrico della derivata, verifica che il valore dell’ordinata d del centro della ruota si mantiene costante durante il moto. Pertanto, al ciclista sembra di muoversi su una superficie piana.

Prerequisiti

  • conoscenza delle proprietà dei triangoli simili
  • conoscere bene la definizione di derivata e quindi inclinazione di una retta
  • conoscenza del teorema di Pitagora.

Sviluppo

Sfruttando la proprietà dei triangoli simili posso dire che:

(23)   \begin{equation*} LM : AM = AL : CL \end{equation*}

(24)   \begin{equation*} \cfrac{LM}{AM}=\cfrac{AL}{CL} \end{equation*}

questo rapporto è proprio il valore della derivata lungo la curva.

(25)   \begin{equation*} AL=f'(x)\cdot CL \end{equation*}

Applico Pitagora al triangolo DLC

(26)   \begin{equation*} AL^{2}+CL^{2}=AC^{2} \end{equation*}

(27)   \begin{equation*} CL=1 \end{equation*}

in quanto metà del lato

(28)   \begin{equation*} AC= d-f(x) \end{equation*}

utilizzando (25), la (27) e la (28) inserendola nella (26) ho:

(29)   \begin{equation*} f'(x)^{2}+1=\left ( d-f(x) \right )^{2} \end{equation*}

Adesso sostituisco i valori della derivata e della funzione ed ho:

(30)   \begin{gather*} \left (-\cfrac{e^x-e^{-x} }{2} \right )^{2}+1=\left (d-\sqrt{2}+\cfrac{e^x+e^{-x}}{2} \right )^2 \\ \cfrac{e^{2x}-2+e^{-2x}}{4}+1=\left (d-\sqrt{2}+\cfrac{e^x+e^{-x}}{2} \right )^2 \\ \cfrac{e^{2x}-2+e^{-2x}+4}{4}=\left (d-\sqrt{2}+\cfrac{e^x+e^{-x}}{2} \right )^2 \\ \cfrac{e^{2x}+2+e^{-2x}}{4}=\left (d-\sqrt{2}+\cfrac{e^x+e^{-x}}{2} \right )^2 \\ \left (\cfrac{e^{x}+e^{-x}}{2} \right )^2 =\left (d-\sqrt{2}+\cfrac{e^x+e^{-x}}{2} \right )^2 \\ \cfrac{e^{x}+e^{-x}}{2} =d-\sqrt{2}+\cfrac{e^x+e^{-x}}{2} \\ 0 =d-\sqrt{2} \\ d =\sqrt{2} \end{gather*}

e quindi d rimane costante.

Anche il grafico della funzione:

f(x)=\cfrac{2}{\sqrt{3} }-\cfrac{e^{x}+e^{-x}}{2} per x\in \left [ -\frac{\ln 3}{2} ; \frac{\ln 3}{2}\right ]
se replicato varie volte, può rappresentare il profilo di una pedana adatta a essere percorsa da una bicicletta con ruote molto particolari, aventi la forma di un poligono regolare.

4) Individua tale poligono regolare, motivando la risposta

Prerequisiti

  • relazione tra derivata ed inclinazione della retta
  • conoscere le proprietà dei logaritmi
  • saper bene le derivate

Sviluppo

L’ipotesi di partenza già mi dice che questo è un nuovo profilo per cui non devo fare tutti i passaggi precedenti e sempre nell’ipotesi gli estremi in cui è definita la funzione rappresentano le intersezioni con gli assi. Si può fare la verifica ma è sicuramente ridondante.

Allora calcolo la derivata prima nei punti d’intersezione.

La derivata prima è sempre la (6) che riporto:

f(x)^{'}=-\cfrac{e^{x}-e^{-x}}{2}

La devo calcolare nei punti d’intersezione:

(31)   \begin{gather*} f'\left (-\cfrac{\ln 3}{2} \right )=-\cfrac{e^ {-\frac{1}{2}\ln 3} -e^{-\frac{1}{2}\ln 3}}{2} \\ -\cfrac{\frac{1}{\sqrt{3}}-\sqrt{3}}{2}= -\cfrac{1-3}{2\sqrt{3}}=-\cfrac{-2}{2\sqrt{3}}=\cfrac{1}{\sqrt{3}} \end{gather*}

in maniera analoga con l’altro punto d’intersezione e l’inclinazione varrà:

(32)   \begin{equation*} f'\left (-\cfrac{\ln 3}{2} \right )=-\cfrac{1}{\sqrt{3}} \end{equation*}

Avere il valore dell’inclinazione della retta è come avere il valore della tangente di cui devo trovare il relativo angolo:

\tan \alpha =\cfrac{1}{\sqrt{3}}

Per cui l’angolo che forma la retta con l’asse delle ascisse vale esattamente

\alpha =30^{0}

Anche l’altra retta forma un angolo di 30^{0} per cui l’angolo tra i lati di questo poligono è 120^{0}

L’unico poligono che presenta tale angolo tra i sui lati è un esagono.[:]

Pubblicato in Senza categoria | Contrassegnato , , | Lascia un commento

[:it]Maturità 2017: quesito 1[:]

[:it]

Definito il numero E come:

(1)   \begin{equation*} E=\int_{0}^{1}xe^{x}dx \end{equation*}

dimostrare che risulta:

(2)   \begin{equation*} \int_{0}^{1}x^{2}e^{x}dx=e-2E \end{equation*}

ed esprimere

(3)   \begin{equation*} \int_{0}^{1}x^{3}e^{x}dx \end{equation*}

in termini di e ed E.

Prerequisiti:

  • conoscere l’integrazione per parti

Sviluppo:

La (2) la si risolve integrando per parti, ed utilizzo il seguente schema:

f(x)=x^{2} g(x)=e^{x}
f^{'}(x)=2x g^{'}(x)=e^{x}

 

(4)   \begin{equation*} \int_{0}^{1}x^{2}e^{x}dx\left=\begin{matrix} x^{2}e^{x}\end{matrix}\right|_{0}^{1}-2\int_{0}^{1}xe^{x}dx= e-2E \end{equation*}

 

Anche la (3) la si risolve per parti ed utilizzo il seguente schema:

f(x)=x^{3} g(x)=e^{x}
f^{'}(x)=3x^2 g^{'}(x)=e^{x}

(5)   \begin{equation*} \int_{0}^{1}x^{3}e^{x}dx\left=\begin{matrix} x^{3}e^{x}\end{matrix}\right|_{0}^{1}-3\int_{0}^{1}x^2e^{x}dx \end{equation*}

ed utilizzando le relazioni precedentemente trovare ho:

(6)   \begin{equation*} \int_{0}^{1}x^{3}e^{x}dx=e-3(e-2E)=e-3e+6E=-2e+6E \end{equation*}

 [:]

Pubblicato in Senza categoria | Contrassegnato , | Lascia un commento